World's most popular travel blog for travel bloggers.

[Solved]: A Reduction from XORSAT to 2-SAT

, , No Comments
Problem Detail: 

Does anyone know of a non-trivial reduction from XORSAT to 2-sat since they are both in P? (By non-trivial I mean one that does not just solve the instance of XORSAT and map it to a fixed instance of 2-sat. Rather I'm looking for a way to solve XORSAT by a different method other than just using Gaussian Elimination or some other method of linear algebra.)

Asked By : Ari

Answered By : Yuval Filmus

Your problem can be posed more formally as follows: Is there a weak reduction from XORSAT to 2SAT? Here weak can be, for example, logspace or AC$^0$.

We know that 2SAT is NL-complete under AC$^0$ reductions, while restricted versions of XORSAT (say 3XORSAT) are $\oplus$L-complete under AC$^0$ reductions, see this paper proving a refined Schaefer dichotomy theorem. Although NL$\subseteq \oplus$L non-uniformly (see this answer), we don't expect the converse to hold, and in particular we don't expect there to be a weak reduction from $\oplus$L to NL. Unfortunately I'm unaware of any concrete results in this direction.

Summarizing, it is expected that there is no weak reduction from XORSAT to 2SAT, but I'm not sure there's much technical work supporting this conjecture.

Best Answer from StackOverflow

Question Source : http://cs.stackexchange.com/questions/28435

3.2K people like this

 Download Related Notes/Documents

0 comments:

Post a Comment

Let us know your responses and feedback